Show that $int fge 3/2$

The name of the pictureThe name of the pictureThe name of the pictureClash Royale CLAN TAG#URR8PPP











up vote
4
down vote

favorite
2












I'm trying to prove a question of my homework, but I'm stuck. The question is




Question: let $f:[0,1]rightarrow mathbbR$, such that $|f'(x)| < 1, forall x$ and $f(0) = 2$, prove that $int_0^1f(x)dx geq frac32$




Ok, so my efforts until now were these:



  1. I know that if $|f'(x)| < 1$, can be proved that $f(x)-f(y) < |x-y|$, (I don't prove that inequality is strictly), so I can use the condition on hypothesis $Rightarrow$ $|f(x)-2| leq |f(x)| -2 < |x|$ $Rightarrow$ $int_0^1f(x)dx < int_0^1x-2dx$, so I only can conclude that $int_0^1f(x)dx < 5/2$.

  2. My other try is to using the mean value theorem so I can write down $f'(c) = fracf(1) - f(0)1-0 <1$, for some $c in (a,b)$, then $f(1) < 3$.

I'm not sure about my first effort, but I'm confident about the second one.



Thank you in advance!







share|cite|improve this question






















  • Do you mean $|f'(x)|<1?$ In other case it is false: $f(x)=2-4x.$
    – mfl
    Aug 15 at 23:46










  • @mfl Exactly my comment, you got there first
    – Rushabh Mehta
    Aug 15 at 23:46










  • Yes, sorry guys
    – Matheus Fachini
    Aug 15 at 23:49














up vote
4
down vote

favorite
2












I'm trying to prove a question of my homework, but I'm stuck. The question is




Question: let $f:[0,1]rightarrow mathbbR$, such that $|f'(x)| < 1, forall x$ and $f(0) = 2$, prove that $int_0^1f(x)dx geq frac32$




Ok, so my efforts until now were these:



  1. I know that if $|f'(x)| < 1$, can be proved that $f(x)-f(y) < |x-y|$, (I don't prove that inequality is strictly), so I can use the condition on hypothesis $Rightarrow$ $|f(x)-2| leq |f(x)| -2 < |x|$ $Rightarrow$ $int_0^1f(x)dx < int_0^1x-2dx$, so I only can conclude that $int_0^1f(x)dx < 5/2$.

  2. My other try is to using the mean value theorem so I can write down $f'(c) = fracf(1) - f(0)1-0 <1$, for some $c in (a,b)$, then $f(1) < 3$.

I'm not sure about my first effort, but I'm confident about the second one.



Thank you in advance!







share|cite|improve this question






















  • Do you mean $|f'(x)|<1?$ In other case it is false: $f(x)=2-4x.$
    – mfl
    Aug 15 at 23:46










  • @mfl Exactly my comment, you got there first
    – Rushabh Mehta
    Aug 15 at 23:46










  • Yes, sorry guys
    – Matheus Fachini
    Aug 15 at 23:49












up vote
4
down vote

favorite
2









up vote
4
down vote

favorite
2






2





I'm trying to prove a question of my homework, but I'm stuck. The question is




Question: let $f:[0,1]rightarrow mathbbR$, such that $|f'(x)| < 1, forall x$ and $f(0) = 2$, prove that $int_0^1f(x)dx geq frac32$




Ok, so my efforts until now were these:



  1. I know that if $|f'(x)| < 1$, can be proved that $f(x)-f(y) < |x-y|$, (I don't prove that inequality is strictly), so I can use the condition on hypothesis $Rightarrow$ $|f(x)-2| leq |f(x)| -2 < |x|$ $Rightarrow$ $int_0^1f(x)dx < int_0^1x-2dx$, so I only can conclude that $int_0^1f(x)dx < 5/2$.

  2. My other try is to using the mean value theorem so I can write down $f'(c) = fracf(1) - f(0)1-0 <1$, for some $c in (a,b)$, then $f(1) < 3$.

I'm not sure about my first effort, but I'm confident about the second one.



Thank you in advance!







share|cite|improve this question














I'm trying to prove a question of my homework, but I'm stuck. The question is




Question: let $f:[0,1]rightarrow mathbbR$, such that $|f'(x)| < 1, forall x$ and $f(0) = 2$, prove that $int_0^1f(x)dx geq frac32$




Ok, so my efforts until now were these:



  1. I know that if $|f'(x)| < 1$, can be proved that $f(x)-f(y) < |x-y|$, (I don't prove that inequality is strictly), so I can use the condition on hypothesis $Rightarrow$ $|f(x)-2| leq |f(x)| -2 < |x|$ $Rightarrow$ $int_0^1f(x)dx < int_0^1x-2dx$, so I only can conclude that $int_0^1f(x)dx < 5/2$.

  2. My other try is to using the mean value theorem so I can write down $f'(c) = fracf(1) - f(0)1-0 <1$, for some $c in (a,b)$, then $f(1) < 3$.

I'm not sure about my first effort, but I'm confident about the second one.



Thank you in advance!









share|cite|improve this question













share|cite|improve this question




share|cite|improve this question








edited Aug 16 at 4:40









Parcly Taxel

33.6k136588




33.6k136588










asked Aug 15 at 23:44









Matheus Fachini

546




546











  • Do you mean $|f'(x)|<1?$ In other case it is false: $f(x)=2-4x.$
    – mfl
    Aug 15 at 23:46










  • @mfl Exactly my comment, you got there first
    – Rushabh Mehta
    Aug 15 at 23:46










  • Yes, sorry guys
    – Matheus Fachini
    Aug 15 at 23:49
















  • Do you mean $|f'(x)|<1?$ In other case it is false: $f(x)=2-4x.$
    – mfl
    Aug 15 at 23:46










  • @mfl Exactly my comment, you got there first
    – Rushabh Mehta
    Aug 15 at 23:46










  • Yes, sorry guys
    – Matheus Fachini
    Aug 15 at 23:49















Do you mean $|f'(x)|<1?$ In other case it is false: $f(x)=2-4x.$
– mfl
Aug 15 at 23:46




Do you mean $|f'(x)|<1?$ In other case it is false: $f(x)=2-4x.$
– mfl
Aug 15 at 23:46












@mfl Exactly my comment, you got there first
– Rushabh Mehta
Aug 15 at 23:46




@mfl Exactly my comment, you got there first
– Rushabh Mehta
Aug 15 at 23:46












Yes, sorry guys
– Matheus Fachini
Aug 15 at 23:49




Yes, sorry guys
– Matheus Fachini
Aug 15 at 23:49










2 Answers
2






active

oldest

votes

















up vote
6
down vote



accepted










You are on the right way. We have that



$$|f(x)-f(0)|le x.$$



Edit



Because of the mean value theorem for $xin (0,1]$ we have that there exist $cin (0,x)$ such that $f(x)-f(0)=f'(c)(x-0).$ Thus we have



$$|f(x)-2|=|f'(c)||x|<|x|.$$



End



Thus, it is



$$2-xle f(x)le 2+x.$$ (Note that the useful inequality here is $2-xle f(x)$ and not $f(x)le 2+x$.)



Integrantig over $[0,1]$ we have



$$2-dfrac12le int_0^1 f(x)dx$$ and we are done.






share|cite|improve this answer






















  • Can you think and way to prove this exercise just using mean value theorem?
    – Matheus Fachini
    Aug 15 at 23:59










  • The first inequality is obtained using the mean value theorem. I have editted the answer to give more details.
    – mfl
    Aug 16 at 0:02

















up vote
1
down vote













fou can also prove it in this way
We know that $f^'(x)>-1$ and $f(0) = 2$



$f(x) = f(0)+int_0^xf^'(t),dt$



so



$f(x)>f(0) + int_0^x-1,dt$



$f(x)>2-x$



$int_0^1f(x)dx >int_0^12-x,dx=1.5$



This image is also an explanation:
Geometric Viewpoint






share|cite|improve this answer






















    Your Answer




    StackExchange.ifUsing("editor", function ()
    return StackExchange.using("mathjaxEditing", function ()
    StackExchange.MarkdownEditor.creationCallbacks.add(function (editor, postfix)
    StackExchange.mathjaxEditing.prepareWmdForMathJax(editor, postfix, [["$", "$"], ["\\(","\\)"]]);
    );
    );
    , "mathjax-editing");

    StackExchange.ready(function()
    var channelOptions =
    tags: "".split(" "),
    id: "69"
    ;
    initTagRenderer("".split(" "), "".split(" "), channelOptions);

    StackExchange.using("externalEditor", function()
    // Have to fire editor after snippets, if snippets enabled
    if (StackExchange.settings.snippets.snippetsEnabled)
    StackExchange.using("snippets", function()
    createEditor();
    );

    else
    createEditor();

    );

    function createEditor()
    StackExchange.prepareEditor(
    heartbeatType: 'answer',
    convertImagesToLinks: true,
    noModals: false,
    showLowRepImageUploadWarning: true,
    reputationToPostImages: 10,
    bindNavPrevention: true,
    postfix: "",
    noCode: true, onDemand: true,
    discardSelector: ".discard-answer"
    ,immediatelyShowMarkdownHelp:true
    );



    );








     

    draft saved


    draft discarded


















    StackExchange.ready(
    function ()
    StackExchange.openid.initPostLogin('.new-post-login', 'https%3a%2f%2fmath.stackexchange.com%2fquestions%2f2884231%2fshow-that-int-f-ge-3-2%23new-answer', 'question_page');

    );

    Post as a guest






























    2 Answers
    2






    active

    oldest

    votes








    2 Answers
    2






    active

    oldest

    votes









    active

    oldest

    votes






    active

    oldest

    votes








    up vote
    6
    down vote



    accepted










    You are on the right way. We have that



    $$|f(x)-f(0)|le x.$$



    Edit



    Because of the mean value theorem for $xin (0,1]$ we have that there exist $cin (0,x)$ such that $f(x)-f(0)=f'(c)(x-0).$ Thus we have



    $$|f(x)-2|=|f'(c)||x|<|x|.$$



    End



    Thus, it is



    $$2-xle f(x)le 2+x.$$ (Note that the useful inequality here is $2-xle f(x)$ and not $f(x)le 2+x$.)



    Integrantig over $[0,1]$ we have



    $$2-dfrac12le int_0^1 f(x)dx$$ and we are done.






    share|cite|improve this answer






















    • Can you think and way to prove this exercise just using mean value theorem?
      – Matheus Fachini
      Aug 15 at 23:59










    • The first inequality is obtained using the mean value theorem. I have editted the answer to give more details.
      – mfl
      Aug 16 at 0:02














    up vote
    6
    down vote



    accepted










    You are on the right way. We have that



    $$|f(x)-f(0)|le x.$$



    Edit



    Because of the mean value theorem for $xin (0,1]$ we have that there exist $cin (0,x)$ such that $f(x)-f(0)=f'(c)(x-0).$ Thus we have



    $$|f(x)-2|=|f'(c)||x|<|x|.$$



    End



    Thus, it is



    $$2-xle f(x)le 2+x.$$ (Note that the useful inequality here is $2-xle f(x)$ and not $f(x)le 2+x$.)



    Integrantig over $[0,1]$ we have



    $$2-dfrac12le int_0^1 f(x)dx$$ and we are done.






    share|cite|improve this answer






















    • Can you think and way to prove this exercise just using mean value theorem?
      – Matheus Fachini
      Aug 15 at 23:59










    • The first inequality is obtained using the mean value theorem. I have editted the answer to give more details.
      – mfl
      Aug 16 at 0:02












    up vote
    6
    down vote



    accepted







    up vote
    6
    down vote



    accepted






    You are on the right way. We have that



    $$|f(x)-f(0)|le x.$$



    Edit



    Because of the mean value theorem for $xin (0,1]$ we have that there exist $cin (0,x)$ such that $f(x)-f(0)=f'(c)(x-0).$ Thus we have



    $$|f(x)-2|=|f'(c)||x|<|x|.$$



    End



    Thus, it is



    $$2-xle f(x)le 2+x.$$ (Note that the useful inequality here is $2-xle f(x)$ and not $f(x)le 2+x$.)



    Integrantig over $[0,1]$ we have



    $$2-dfrac12le int_0^1 f(x)dx$$ and we are done.






    share|cite|improve this answer














    You are on the right way. We have that



    $$|f(x)-f(0)|le x.$$



    Edit



    Because of the mean value theorem for $xin (0,1]$ we have that there exist $cin (0,x)$ such that $f(x)-f(0)=f'(c)(x-0).$ Thus we have



    $$|f(x)-2|=|f'(c)||x|<|x|.$$



    End



    Thus, it is



    $$2-xle f(x)le 2+x.$$ (Note that the useful inequality here is $2-xle f(x)$ and not $f(x)le 2+x$.)



    Integrantig over $[0,1]$ we have



    $$2-dfrac12le int_0^1 f(x)dx$$ and we are done.







    share|cite|improve this answer














    share|cite|improve this answer



    share|cite|improve this answer








    edited Aug 16 at 0:02

























    answered Aug 15 at 23:50









    mfl

    24.7k12040




    24.7k12040











    • Can you think and way to prove this exercise just using mean value theorem?
      – Matheus Fachini
      Aug 15 at 23:59










    • The first inequality is obtained using the mean value theorem. I have editted the answer to give more details.
      – mfl
      Aug 16 at 0:02
















    • Can you think and way to prove this exercise just using mean value theorem?
      – Matheus Fachini
      Aug 15 at 23:59










    • The first inequality is obtained using the mean value theorem. I have editted the answer to give more details.
      – mfl
      Aug 16 at 0:02















    Can you think and way to prove this exercise just using mean value theorem?
    – Matheus Fachini
    Aug 15 at 23:59




    Can you think and way to prove this exercise just using mean value theorem?
    – Matheus Fachini
    Aug 15 at 23:59












    The first inequality is obtained using the mean value theorem. I have editted the answer to give more details.
    – mfl
    Aug 16 at 0:02




    The first inequality is obtained using the mean value theorem. I have editted the answer to give more details.
    – mfl
    Aug 16 at 0:02










    up vote
    1
    down vote













    fou can also prove it in this way
    We know that $f^'(x)>-1$ and $f(0) = 2$



    $f(x) = f(0)+int_0^xf^'(t),dt$



    so



    $f(x)>f(0) + int_0^x-1,dt$



    $f(x)>2-x$



    $int_0^1f(x)dx >int_0^12-x,dx=1.5$



    This image is also an explanation:
    Geometric Viewpoint






    share|cite|improve this answer


























      up vote
      1
      down vote













      fou can also prove it in this way
      We know that $f^'(x)>-1$ and $f(0) = 2$



      $f(x) = f(0)+int_0^xf^'(t),dt$



      so



      $f(x)>f(0) + int_0^x-1,dt$



      $f(x)>2-x$



      $int_0^1f(x)dx >int_0^12-x,dx=1.5$



      This image is also an explanation:
      Geometric Viewpoint






      share|cite|improve this answer
























        up vote
        1
        down vote










        up vote
        1
        down vote









        fou can also prove it in this way
        We know that $f^'(x)>-1$ and $f(0) = 2$



        $f(x) = f(0)+int_0^xf^'(t),dt$



        so



        $f(x)>f(0) + int_0^x-1,dt$



        $f(x)>2-x$



        $int_0^1f(x)dx >int_0^12-x,dx=1.5$



        This image is also an explanation:
        Geometric Viewpoint






        share|cite|improve this answer














        fou can also prove it in this way
        We know that $f^'(x)>-1$ and $f(0) = 2$



        $f(x) = f(0)+int_0^xf^'(t),dt$



        so



        $f(x)>f(0) + int_0^x-1,dt$



        $f(x)>2-x$



        $int_0^1f(x)dx >int_0^12-x,dx=1.5$



        This image is also an explanation:
        Geometric Viewpoint







        share|cite|improve this answer














        share|cite|improve this answer



        share|cite|improve this answer








        edited Aug 16 at 0:55









        Clayton

        18.3k22883




        18.3k22883










        answered Aug 16 at 0:50









        Xiangwei Wang

        714




        714






















             

            draft saved


            draft discarded


























             


            draft saved


            draft discarded














            StackExchange.ready(
            function ()
            StackExchange.openid.initPostLogin('.new-post-login', 'https%3a%2f%2fmath.stackexchange.com%2fquestions%2f2884231%2fshow-that-int-f-ge-3-2%23new-answer', 'question_page');

            );

            Post as a guest













































































            這個網誌中的熱門文章

            Is there any way to eliminate the singular point to solve this integral by hand or by approximations?

            Why am i infinitely getting the same tweet with the Twitter Search API?

            Carbon dioxide